Difference between revisions of "2002 AMC 10P Problems/Problem 23"

(Created page with "== Solution 1== == See also == {{AMC10 box|year=2002|ab=P|num-b=22|num-a=24}} {{MAA Notice}}")
 
(Problem)
 
(One intermediate revision by the same user not shown)
Line 1: Line 1:
 +
== Problem ==
 +
 +
Let
 +
<cmath>a=\frac{1^2}{1} + \frac{2^2}{3} + \frac{3^2}{5} +  \; \dots \; + \frac{1001^2}{2001}</cmath>
 +
 +
and
 +
 +
<cmath>b=\frac{1^2}{3} + \frac{2^2}{5} + \frac{3^2}{7} +  \; \dots \; + \frac{1001^2}{2003}.</cmath>
 +
 +
Find the integer closest to <math>a-b.</math>
 +
 +
<math>
 +
\text{(A) }500
 +
\qquad
 +
\text{(B) }501
 +
\qquad
 +
\text{(C) }999
 +
\qquad
 +
\text{(D) }1000
 +
\qquad
 +
\text{(E) }1001
 +
</math>
 +
 
== Solution 1==
 
== Solution 1==
  

Latest revision as of 22:18, 14 July 2024

Problem

Let \[a=\frac{1^2}{1} + \frac{2^2}{3} + \frac{3^2}{5} +  \; \dots \; + \frac{1001^2}{2001}\]

and

\[b=\frac{1^2}{3} + \frac{2^2}{5} + \frac{3^2}{7} +  \; \dots \; + \frac{1001^2}{2003}.\]

Find the integer closest to $a-b.$

$\text{(A) }500 \qquad \text{(B) }501 \qquad \text{(C) }999 \qquad \text{(D) }1000 \qquad \text{(E) }1001$

Solution 1

See also

2002 AMC 10P (ProblemsAnswer KeyResources)
Preceded by
Problem 22
Followed by
Problem 24
1 2 3 4 5 6 7 8 9 10 11 12 13 14 15 16 17 18 19 20 21 22 23 24 25
All AMC 10 Problems and Solutions

The problems on this page are copyrighted by the Mathematical Association of America's American Mathematics Competitions. AMC logo.png